I was watching the explanation for the in/out game referring to a group of people who can only be hired if they are interviewed for a position. In the explanation for the last question in the set, it mentions that the problem states at ...
Hi all - I'd really appreciate your help on understanding the argument in this question.
I get the gap in this question is that just because first doctrine states that "all historical events must be explained in economic factors" doesn't ...
Can someone help explain this question to me? It's the first LR question I haven't been able to understand, even after blind review and review. I chose answer choice B.
I do not even know where to begin with the ACs.
I understand the stimulus as a theater recently closed because the owner claimed that it could not regularly sustain an audience. Then a week prior to the closure, massive profit resulted.
None ...
Personal Injury (PI) or personal damage exceeding 500 (PD>500)------> LR (legally required to report accident)
"Unless the driver is incapable"- Capable-----> /LR (is this right?)
/LR (Ted is NOT legally required to report accident)< ...
Facts:
1.) We found titanium in 2 Bibles: (JG and B-36)
2.) Titanium in 15th century Vinland Map (purportedly)
Conclusion: B-36 must have been printed by JG and Vinland Map is from 15th century
Flaw:
Is it wrong for the ...
I posted this under the Q-specific help vid, but the discussion forum here seems to get more attention sometimes, so double-posting. Promise to add helpful responses I get to my original discussion for our future LSAT progeny :3< ...
The question stem asks: "The reasoning in the journalist's argument is most vulnerable to criticism on the grounds that the argument fails to consider that..." and the correct answer, E, says the flaw is that those who donate might not be those who join ...
So for 24, I was confused by the question stem. When it says support the position of Passage B over that of Passage A, does that mean we are trying to strengthen B and weaken A? Or just strengthen B and have neutral effect on A? Confusing AF. And I got ...
For this question, I did not choose A because I created a world where F is not 1 lower than S and I don't understand what I'm getting wrong, I doubled checked the rules.
The world is: F K J S M G H
lol I know I did not crack the lsat, # ...
Could someone please help explain to me how "C" is the correct answer? I went with choice E because it seemed like the closest to correct, but I honestly didn't think any of them seemed right. I'm reviewing and still can't figure out how that is the right ...
... , by accident, that PT 26 S1 Game 2 and PT 81 S4 Game ... because I had randomly done PT 26 S1 game 2 as a ... warmup LG before taking PT 81 today ...
... in PTs. For example, PT45 was originally released with two ... questions to Drill with. PT 76's second LR ... Modern" and take PTs 45 and higher, there will ... absolutely no overlap between PT LR and Drilling pool ... to have a fresh PT with a reliable PT score. ...